Difference between revisions of "MIE 2016/Problem 2"
(→See Also) |
|||
Line 15: | Line 15: | ||
(e) <math>k\geq8</math> | (e) <math>k\geq8</math> | ||
− | ==Solution 2== | + | ==Solution 1== |
+ | ===Objective:=== | ||
+ | We can solve this problem in two steps: First, we solve for the range of <math>\frac{x^2-2x-14}{x}>3</math>, then combine it with the range of <math>x\leq12</math> to get a compound inequality which we can use to find all possible integer solutions. | ||
+ | ===Step 1=== | ||
+ | We first find the range of the inequality <math>\frac{x^2-2x-14}{x}>3</math>. | ||
+ | We now simplify the inequality: | ||
+ | |||
+ | Case 1: <math>0<x\leq12</math> | ||
+ | |||
+ | <cmath>\frac{x^2-2x-14}{x}>3</cmath> | ||
+ | <cmath>x^2 - 2x - 14>3x</cmath> | ||
+ | <cmath>x^2 - 5x - 14 > 0</cmath> | ||
+ | Factoring, we get | ||
+ | <cmath>(x-7)(x+2)>0</cmath> | ||
+ | Now, <math>x</math> can be greater than <math>7</math> or less than <math>-2</math>. | ||
+ | But in this case, <math>0<x\leq12</math>, and this further restricts our solutions. | ||
+ | So, for the case where <math>0<x\leq12</math>, our solutions are <math>7<x\leq12</math> | ||
+ | |||
+ | Case 2: <math>x<0</math> | ||
+ | <cmath>\frac{x^2-2x-14}{x}>3</cmath> | ||
+ | <cmath>x^2-2x-14<3x</cmath> | ||
+ | <cmath>x^2-5x - 14<0</cmath> | ||
+ | <cmath>(x-7)(x+2)<0</cmath> | ||
+ | We have in this case that <math>-2<x<7</math>, but the case statement further restricts our solutions. | ||
+ | |||
+ | For this case, the solutions are <math>-2<x<0</math> | ||
+ | |||
+ | ===Step 2=== | ||
+ | Now, we know the solutions for <math>x</math>: in the first case, where <math>7<x\leq12</math>, the integer solutions are <math>x = {8, 9, 10, 11, 12}</math> | ||
+ | |||
+ | In the second case, where <math>-2<x<0</math>, the only integer solution is <math>x = {-1}</math> | ||
+ | |||
+ | The union of these two cases gives <math>x = {-1, 8, 9, 10, 11, 12}</math>. | ||
+ | |||
+ | There are <math>k=6</math> solutions and <math>6\leq k\leq8</math>, giving <math>\boxed{D)}</math> | ||
==See Also== | ==See Also== |
Revision as of 10:07, 10 September 2020
Problem 2
The following system has integer solutions. We can say that:
(a)
(b)
(c)
(d)
(e)
Solution 1
Objective:
We can solve this problem in two steps: First, we solve for the range of , then combine it with the range of to get a compound inequality which we can use to find all possible integer solutions.
Step 1
We first find the range of the inequality .
We now simplify the inequality:
Case 1:
Factoring, we get Now, can be greater than or less than . But in this case, , and this further restricts our solutions. So, for the case where , our solutions are
Case 2: We have in this case that , but the case statement further restricts our solutions.
For this case, the solutions are
Step 2
Now, we know the solutions for : in the first case, where , the integer solutions are
In the second case, where , the only integer solution is
The union of these two cases gives .
There are solutions and , giving